Diễn Đàn MathScopeDiễn Đàn MathScope
  Diễn Đàn MathScope
Ghi Danh Hỏi/Ðáp Thành Viên Social Groups Lịch Ðánh Dấu Ðã Ðọc

Go Back   Diễn Đàn MathScope > Sơ Cấp > Việt Nam và IMO > 2018

News & Announcements

Ngoài một số quy định đã được nêu trong phần Quy định của Ghi Danh , mọi người tranh thủ bỏ ra 5 phút để đọc thêm một số Quy định sau để khỏi bị treo nick ở MathScope nhé !

* Nội quy MathScope.Org

* Một số quy định chung !

* Quy định về việc viết bài trong diễn đàn MathScope

* Nếu bạn muốn gia nhập đội ngũ BQT thì vui lòng tham gia tại đây

* Những câu hỏi thường gặp

* Về việc viết bài trong Box Đại học và Sau đại học


Trả lời Gởi Ðề Tài Mới
 
Ðiều Chỉnh Xếp Bài
Old 10-02-2018, 04:22 PM   #16
namdung
Administrator

 
Tham gia ngày: Feb 2009
Đến từ: Tp Hồ Chí Minh
Bài gởi: 1,343
Thanks: 209
Thanked 4,066 Times in 778 Posts
Gửi tin nhắn qua Yahoo chát tới namdung
Đề 2 - Ngày 2, 10/2/2018


4. Tồn tại hay không số nguyên dương $a$ sao cho với mọi $ m;\, n\in\mathbb Z^{+} , \;m;\, n> a $ thì luôn chia được các hình chữ nhật $m×n$ thành các hình chữ nhật $ 4\times 6 $ và $ 5\times 7 $.

5. Tìm tất cả các hàm số $f$ xác định trên tập hợp các số hữu tỷ dương và nhận giá trị trên tập hợp đó thỏa mãn đồng thời các điều kiện sau:
  1. $ f(x) + f\left(\dfrac{1}{x}\right) = 1 $ với mọi $x$ thuộc $\mathbb Q^{+}$.
  2. $f(2x+1) = \dfrac{1}{2}f(x) $ với mọi $x$ thuộc $Q^{+}$.

6. Tứ giác $ABCD$ ngoại tiếp đường tròn $(\omega)$. Gọi $I$ và $J$ là tâm đường tròn nội tiếp các tam giác $ABC$ và $ADC$. Ta dựng được hai đường tròn cùng đi qua $A$ và $C$ và tiếp xúc với $(\omega)$. Gọi các tiếp điểm lần lượt là $K$ và $L$. Chứng minh rằng bốn điểm $I,\,J,\, K,\, L$ cùng nằm trên một đường tròn.


PS. File pdf tổng hợp đề 2, download bên dưới.
[RIGHT][I][B]Nguồn: MathScope.ORG[/B][/I][/RIGHT]
 
File Kèm Theo
Kiểu File : pdf 2tst2018r2.pdf (86.4 KB, 55 lần tải)
namdung is offline   Trả Lời Với Trích Dẫn
Old 10-02-2018, 06:59 PM   #17
namdung
Administrator

 
Tham gia ngày: Feb 2009
Đến từ: Tp Hồ Chí Minh
Bài gởi: 1,343
Thanks: 209
Thanked 4,066 Times in 778 Posts
Gửi tin nhắn qua Yahoo chát tới namdung
Tóm tắt lời giải đề 1, ngày 2.

Bài 4 hơi kỹ thuật một chút, tôi sẽ post lời giải sau một chút.

Bài 5 có một lời giải và bình luận của bạn pco, nhưng có vẻ vẫn còn nhiều ý phải bàn, mặc dùng ý tưởng là rất thú vị.

Có thể tóm tắt lời giải như sau:
1) Chia hình vuông thành n hình chữ nhật con bằng các đường thẳng dọc, mỗi hình chứa n điểm (các điểm có thể nằm trên biên). Trong mỗi hình chữ nhật con, ta nối các điểm lần lượt theo tung độ tăng dần, thành $n$ xích, mỗi xích có $n$ điểm, $n-1$ đoạn.
2) Đánh giá tổng độ dài n xích này đơn giản: Tổng hình chiếu mỗi xích lên trục tung không vượt quá 1, tổng hình chiếu mỗi xích lên trục hoành không quá $(n-1)h_i$, với $h_i$ là chiều rộng của hình chữ nhật thứ $i$. Do đó tổng các hình chiếu theo trục tung của và trục hoành của $n$ xích tương ứng không vượt quá $n$ và $n-1$.
3) Khó nhất là xử lý các đoạn nối. Ta có hai cách nối các xích này với nhau (trên-dưới-trên-dưới ... hoặc dưới - trên - dưới - trên). Tổng hình chiếu (lên trục hoành) các đoạn nối này bằng tổng hình chiếu của đường gấp khúc nối những điểm có tung độ lớn nhất và đường gấp khúc nối những điểm có tung độ nhỏ nhất, do đó nhỏ hơn hay bằng $2$. Vì vậy tồn tại một cách nối có tổng hình chiếu các đoạn nối lên trục hoành nhỏ hơn hay bằng $1$.
4) Với hình chiếu lên trục tung, ta có thể "ghép" các đoạn nối này vào một trong hai xích mà nó nối để đánh giá, để không tạo thêm bất kỳ một tổng nào nữa.

Bài 6 được giải dựa vào lý thuyết phương trình Pell. Xem lời giải của bạn chienthan và phần giải thích bổ sung của bạn muaxl2xo.
[RIGHT][I][B]Nguồn: MathScope.ORG[/B][/I][/RIGHT]
 
namdung is offline   Trả Lời Với Trích Dẫn
Old 18-02-2018, 11:48 PM   #18
sternritterp28
+Thành Viên+
 
Tham gia ngày: Aug 2015
Bài gởi: 4
Thanks: 1
Thanked 0 Times in 0 Posts
Trích:
Nguyên văn bởi namdung View Post
[INDENT]
Đề 2 - Ngày 2, 10/2/2018

5. Tìm tất cả các hàm số $f$ xác định trên tập hợp các số hữu tỷ dương và nhận giá trị trên tập hợp đó thỏa mãn đồng thời các điều kiện sau:
  1. $ f(x) + f\left(\dfrac{1}{x}\right) = 1 $ với mọi $x$ thuộc $\mathbb Q^{+}$.
  2. $f(2x+1) = \dfrac{1}{2}f(x) $ với mọi $x$ thuộc $Q^{+}$.
Thay $x=1$ vào (i), dễ có $f(1) = \dfrac{1}{2}$. Thế liên tục vào bước 2 và quy nạp, $f(2^k-1) = \dfrac{1}{2^k}$

Ta xét $x = \dfrac{1}{p+1} = \dfrac{a}{b}$ ($a, b \in N, p \in Q^{+}, a, b$ lẻ, $a < b, (a; b) = 1$).
Ta đưa ra 2 dãy $(p_n) và (x_n)$ như sau:
$(p_n)$: \begin{align}
\begin{cases}
p_0 &= p \\
p_{n+1} &= \dfrac{p_n-1}{2} (p_n > 1) \\
p_{n+1} &= \dfrac{1}{p_n} (p_n < 1)
\end{cases}
\end{align}
$(x_n)$: \begin{align}
\begin{cases}
x_0 &= x \\
x_{n+1} &= 2x_n (x_n < \dfrac{1}{2}) \\
x_{n+1} &= 1 - x_n (x_n > \dfrac{1}{2})
\end{cases}
\end{align}

Ta chứng minh 2 ý sau:
(i) $x_n = \dfrac{1}{p_n + 1} \forall n \in N$ (dễ kiểm chứng trực tiếp).
(ii) $\exists n_0 > 0: p_n = p_0$ (dãy $(p_n)$ tuần hoàn, từ đó suy ra $(x_n)$ tuần hoàn cùng chu kỳ).
CM (ii):
Gọi bước $x_n \rightarrow 2x_n (p_n \rightarrow \dfrac{p_n-1}{2})$ là bước “thuận”, $x_n \rightarrow 1 - x_n (p_n \rightarrow \dfrac{1}{p_n})$ là bước “nghịch”. Hiển nhiên giữa 2 bước “nghịch” liên tiếp sẽ có ít nhất 1 bước “thuận”. Từ $x_n \rightarrow x_{n+1}$, nếu là bước “nghịch” thì hiển nhiên $x_{n+1}$ (dạng tối giản) có mẫu là $b$, tử là số nguyên lẻ. Gọi dãy kết quả của mỗi bước “nghịch” là $x_0, x_{n_1}, x_{n_2},...$ (dãy bắt đầu từ $x_0$) thì mỗi giá trị $x_{n_i}$ xác định đúng một $x_{n_{i-1}}$ và một $x_{n_{i+1}}$, do đó đây là dãy tuần hoàn (do các giá trị nhận được là hữu hạn trong tập {$\dfrac{1}{b}; \dfrac{3}{b}; …; \dfrac{b-2}{b}$})
Vậy $\exists a_1, a_2, …, a_k \in N^{*}: x = 1 - 2^{a_1}(1- 2^{a_2}(... -2^{a_{k-1}}(1-2^{a_k}x)...))$
Theo quy luật trên, $f(p_{n+1}) = \dfrac{f(p_n)}{2}$ (thuận); $f(p_{n+1}) = 1 - f(p_n)$ (nghịch)
Như vậy, $f(p)$ cũng thỏa mãn $f(p) = 1 - 2^{a_1}(1- 2^{a_2}(... -2^{a_{k-1}}(1-2^{a_k}f(p))...))$. Từ đây, $f(p) = x = \dfrac{1}{p+1} \forall p$ thỏa mãn.

Với $p \in Q^{+}$ sao cho $x = \dfrac{1}{p+1}$ không thỏa:
Nếu $x$ (dạng tối giản) có tử chẵn, mẫu lẻ thì $x = \dfrac{2^k.a}{b}$ ($a, b$ lẻ, $2^k.a < b, (a; b) = 1$ thì xét dãy $(x_n), (p_n)$ như trên, sau đó thực hiện bước "thuận" cho đến khi thực hiện lần đầu bước "nghịch", ta có một phân số thỏa điều kiện trên. Truy ngược lại, ta cũng có $f(p) = \dfrac{1}{p+1}$. Với $x$ (dạng tối giản) có tử lẻ, mẫu chẵn thì ta cũng làm tương tự.

Kết luận: $f(x) = \dfrac{1}{x+1} \forall x \in Q^{+} \blacksquare$
[RIGHT][I][B]Nguồn: MathScope.ORG[/B][/I][/RIGHT]
 
sternritterp28 is offline   Trả Lời Với Trích Dẫn
Old 21-02-2018, 11:42 AM   #19
namdung
Administrator

 
Tham gia ngày: Feb 2009
Đến từ: Tp Hồ Chí Minh
Bài gởi: 1,343
Thanks: 209
Thanked 4,066 Times in 778 Posts
Gửi tin nhắn qua Yahoo chát tới namdung
Đã hết Tết rồi, hôm nay chúng ta quay trở lại với Vietnam TST 2018 Preparation.

Ngày mai chúng tôi sẽ post Ngày 1 của đề 3. Hôm nay chúng tôi gửi tóm tắt lời giải Ngày 1 của đề 2.

Bài hình được phát triển từ 1 bài toán của USA MO (năm 2006). Ý tưởng cơ bản là dùng điểm Miquel và tâm vị tự quay. Hy vọng sẽ có bạn nào đó gửi lời giải chi tiết.

Bài tổ hợp là một bài toán sử dụng đơn biến khá thú vị. Chú ý rằng với 3 đội bất kỳ, chỉ có 2 trường hợp xảy ra: hoặc là có 1 đội thắng cả hai trận hoặc là 3 đội này thắng vòng tròn lẫn nhau. Như vậy để đếm số bộ các đội thắng vòng tròn, ta chỉ cần lấy $C_n^3$ trừ đi số bộ mà có 1 đội thắng 2 đội khác. Gọi $p_i$ là số trận thắng của đội thứ $i$ thì số bộ như vậy bằng $\sum_{i=1}^n C^2_{p_i}$. Vì tổng $\sum_{i=1}^n p_i = n(n-1)/2 $ không đổi nên bài toán quy về việc tìm min, max của $\sum_{i=1}^n p_i^2$.
Đến đây ta sử dụng bất biến và đơn biến. Trong khi $\sum_{i=1}^n p_i $ là bất biến thì $\sum_{i=1}^n p_i^2$ là đơn biến qua phép biến đổi $(a, a) \rightarrow (a-1,a+1)$ (mỗi lần như vậy tổng sẽ tăng lên 2 đơn vị, tương ứng tổng $\sum_{i=1}^n C^2_{p_i}$ tăng 1 đơn vị. Từ đó tìm được min của tổng các bình phương đạt được ở trạng thái cân bằng nhất (nếu $n = 2k+1$ thì đó là trạng thái $(k, k, ...., k)$ còn nếu $n = 2k$ thì đó là trạng thái $(k-1,k-1,...,k-1,k, ..., k)$). Điều thú vị là sử dụng nguyên lý Dirichlet, ta thấy rằng ngoại trừ ở trạng thái $(0, 1, 2, ..., n-1)$ thì ở tất cả các trạng thái khác, ta đều tìm được 2 đội bằng điểm và vì thế còn thực hiện được phép biến đổi nói trên để tăng tổng bình phương lên.

Bài 3 ta sử dụng quy nạp và sai phân. Ta phát biểu mệnh đề tổng quát với $2018$ thay bằng $n$. Sau đó ta sử dụng quy nạp theo $n$ với chú ý rằng hết $P(x)$ có bậc $n$ thì $P(x+1) - P(x)$ có bậc $n-1$.
[RIGHT][I][B]Nguồn: MathScope.ORG[/B][/I][/RIGHT]
 
namdung is offline   Trả Lời Với Trích Dẫn
The Following User Says Thank You to namdung For This Useful Post:
hoangleo963 (21-02-2018)
Old 22-02-2018, 03:03 PM   #20
namdung
Administrator

 
Tham gia ngày: Feb 2009
Đến từ: Tp Hồ Chí Minh
Bài gởi: 1,343
Thanks: 209
Thanked 4,066 Times in 778 Posts
Gửi tin nhắn qua Yahoo chát tới namdung
Đề 3 - Ngày 1, 22/2/2018
1. Cho đa giác đều $2017$ cạnh. Ta tô màu các đỉnh bởi 3 màu xanh, đỏ, vàng, với số lượng tương ứng là $a, b, c$ trong đó $a, b, c$ là các số tự nhiên lẻ. Chứng minh rằng tồn tại một tam giác cân có đỉnh là đỉnh của đa giác chứa đủ ba màu xanh, đỏ, vàng.

2. Cho tam giác $ABC$ nhọn, không cân nội tiếp đường tròn $(O)$. Gọi $BE, CF$ là hai đường cao và $H$ là trực tâm. Gọi $d$ là đường thẳng đi qua $H$ và song song với $BC$. Các đường thẳng đối xứng với $BE, CF$ qua $d$ cắt $AB, AC$ lần lượt tại $N, M$. $FM$ cắt $BE$ tại $X$, $EN$ cắt $CF$ tại $Y$. Chứng minh rằng đường tròn ngoại tiếp tam giác $HXY$ tiếp xúc với $(O)$ và đường tròn ngoại tiếp tam giác $HBC$.

3. Cho trước số nguyên dương $m>1$, chứng minh rằng tồn tại một đa thức hệ số nguyên $P(x)$ bất khả quy trên $\mathbb Z [x]$, có bậc $2018$ và có một nghiệm thực là $r$ thỏa mãn $m\mid \left( {1 + \left\lfloor {{r^n}} \right\rfloor } \right)$ với mọi số nguyên dương $n$.

[RIGHT][I][B]Nguồn: MathScope.ORG[/B][/I][/RIGHT]
 
File Kèm Theo
Kiểu File : pdf 2tst2018 (1).pdf (82.0 KB, 49 lần tải)

thay đổi nội dung bởi: namdung, 22-02-2018 lúc 04:47 PM Lý do: Sửa chi tiết bài 2
namdung is offline   Trả Lời Với Trích Dẫn
The Following User Says Thank You to namdung For This Useful Post:
hoangleo963 (22-02-2018)
Old 22-02-2018, 09:54 PM   #21
chienthan
+Thành Viên+
 
Tham gia ngày: Feb 2017
Bài gởi: 11
Thanks: 9
Thanked 8 Times in 6 Posts
Về bài 4 đề số 1. Mọi người có thể xem lời giải bài số 4 trong file này.

Bài này lấy ý tưởng từ BĐT Vasile Cirtoaje. Đáp số câu b là $27$ và $49.$
[RIGHT][I][B]Nguồn: MathScope.ORG[/B][/I][/RIGHT]
 
File Kèm Theo
Kiểu File : pdf Dap An Chu De 1.pdf (497.2 KB, 132 lần tải)
chienthan is offline   Trả Lời Với Trích Dẫn
The Following 2 Users Say Thank You to chienthan For This Useful Post:
kimlinh (17-03-2018), namdung (23-02-2018)
Old 24-02-2018, 10:17 AM   #22
theunknown
+Thành Viên+
 
Tham gia ngày: Jan 2017
Bài gởi: 5
Thanks: 0
Thanked 5 Times in 2 Posts
Bài 6 ngày 2: (em xin lỗi vì không có hình)

Bổ đề: $IJ$ vuông góc với $AC$.
Chứng minh: Gọi $P$ là chân đường vuông góc hạ từ $I$ xuống $AC$. Theo định lý Carnot thì $IA^2-IC^2=PA^2-PC^2=AC.(PA-PC)=AC.(AB-BC)$.
Tương tự ta có $JA^2-JC^2=AC.(DA-DC)$. Do tứ giác $ABCD$ ngoại tiếp nên $AB+DC=BC+AD$, suy ra $IA^2-IC^2=JA^2-JC^2$. Theo định lý Carnot thì $IJ$ vuông góc $AC$.

Trở lại bài toán, không giảm tổng quát ta giả sử $K,L$ lần lượt nằm trong tam giác $ABC,ADC$. Gọi giao điểm của $AC,BD$ là $X$. $M,N$ là tâm đường tròn bàng tiếp góc $B,D$ của tam giác $ABC,ADC$.
Áp dụng định lý Monge D'Alembert cho ba đường tròn $(\omega), (I),(N)$, ta thấy tâm vị tự ngoài của $(I),(N)$ nằm trên $BD$. Hơn nữa do $AC$ là tiếp tuyến chung của $(I),(N)$ và hai đường tròn cùng nằm trên một nửa mặt phẳng bờ $AC$ nên tâm vị tự ngoài của hai đường tròn này cũng nằm trên $AC$. Nói cách khác $X,I,N$ thẳng hàng. Tương tự cũng có $X,J,M$ thẳng hàng.
Áp dụng định lý Protassov, ta có $K,J,M$ thẳng hàng và cùng nằm trên phân giác góc $AKC$, suy ra $KX$ là phân giác $AKC$. Tương tự $LX$ là phân giác $ALC$.
Gọi $Y$ là điểm trên $AC$ mà $(YXAC)=-1$. Gọi giao điểm $IJ$ và $AC$ là $Z$. Khi đó do $KX$ là phân giác góc $AKC$ nên dễ suy ra góc $YKX$ là góc vuông và tương tự góc $YLX$ cũng là góc vuông. Để ý rằng $IJ$ vuông góc $AC$ nên suy ra $YKZJ,YLZI$ là các tứ giác nội tiếp. Từ đó ta có $XK.XJ=XZ.XY=XL.XI$, từ đây suy ra $K,L,I,J$ nằm trên một đường tròn.
[RIGHT][I][B]Nguồn: MathScope.ORG[/B][/I][/RIGHT]
 
theunknown is offline   Trả Lời Với Trích Dẫn
The Following User Says Thank You to theunknown For This Useful Post:
hoangleo963 (25-02-2018)
Old 24-02-2018, 11:35 AM   #23
namdung
Administrator

 
Tham gia ngày: Feb 2009
Đến từ: Tp Hồ Chí Minh
Bài gởi: 1,343
Thanks: 209
Thanked 4,066 Times in 778 Posts
Gửi tin nhắn qua Yahoo chát tới namdung
Đề 3 - Ngày 2, 24/2/2018
4. Trên mặt phẳng cho một số hữu hạn các đường thẳng tô màu xanh hoặc đỏ và không song song. Biết rằng giao điểm của hai đường thẳng cùng màu sẽ nằm trên đường thẳng khác màu hai đường thẳng này. Chứng minh rằng tất cả các đường thẳng này đồng quy.

5. Tìm tất cả các cặp số nguyên tố $p;\, q$ sao cho $a^{3pq} \equiv a \pmod{3pq} $ với mọi số nguyên dương $a$.

6. Cho số nguyên $n \ge 2$. Dãy số dương $\left\{a_k\right\}_{k=1}^n$ được gọi là siêu tăng nếu thỏa mãn điều kiện $$ a_k \ge a_{k-1} +... + a_1 \quad\forall\,k:\;2 \le k \le n.$$ Tìm giá trị lớn nhất của tổng $\sum\limits_{k = 1}^{n - 1} {\dfrac{{{a_k}}}{{{a_{k + 1}}}}} $ với $\left\{a_k\right\}_{k=1}^n$ là một dãy siêu tăng.
PS. File pdf tổng hợp đề 3, download bên dưới.
[RIGHT][I][B]Nguồn: MathScope.ORG[/B][/I][/RIGHT]
 
File Kèm Theo
Kiểu File : pdf 2tst2018r3 (3).pdf (87.5 KB, 44 lần tải)
namdung is offline   Trả Lời Với Trích Dẫn
The Following 3 Users Say Thank You to namdung For This Useful Post:
buratinogigle (01-03-2018), hoangleo963 (25-02-2018), kimlinh (17-03-2018)
Old 25-02-2018, 11:35 AM   #24
sternritterp28
+Thành Viên+
 
Tham gia ngày: Aug 2015
Bài gởi: 4
Thanks: 1
Thanked 0 Times in 0 Posts
Trích:
Nguyên văn bởi namdung View Post

5. Tìm tất cả các cặp số nguyên tố $p;\, q$ sao cho $a^{3pq} \equiv a \pmod{3pq} $ với mọi số nguyên dương $a$.

Em xin chém trước bài này, không biết có sai không

Ta có: $a^{3pq} \equiv a \pmod{3pq}
\Rightarrow a \equiv (a^3)^{pq} \equiv a^{pq} \pmod {3}
\Leftrightarrow pq$ lẻ $\Leftrightarrow p, q \ge 3$.

Xét $p = q \Rightarrow a^{3p^2} \equiv a \pmod{p^2} \Rightarrow a^{3p^2} \equiv a \pmod{p}$.
Chọn $a: (a; p) = 1, ord_p(a) = p - 1 \Rightarrow (3p^2 - 1) \vdots (p - 1) \Rightarrow p = 3$ (do $p$ lẻ)
$\Rightarrow a^{27} \equiv a \pmod{27}$. Thế $a = 2$, loại. Vậy $p \ne q$. Wlog, $3 \le p < q$.

Giả sử $p = 3$. Khi đó $a^{9q} \equiv a \pmod{9q} \Rightarrow a \equiv a^{9p} \equiv a^9 \pmod{p}$.
Lần lượt thay $a = 2, 3 \Rightarrow q = 5 \Rightarrow a^{45} \equiv a \pmod{45}$.
Thay $a = 2: 2^{45} \equiv (2^{10})^4.2^5 \equiv (-2)^4.2^5 \equiv 2^9 \ne 2 \pmod{9} \Rightarrow$ loại.
Vậy $5 \le p < q \Rightarrow q \ge p+2 \ge 7$.

Ta có: $a^{3pq} \equiv a \pmod{pq} \Leftrightarrow$
\begin{align}
\begin{cases}
a \equiv (a^{3p})^q \equiv a^{3p} \pmod{q} \\
a \equiv (a^{3q})^p \equiv a^{3q} \pmod{p}
\end{cases}
\end{align}
Chọn $a: (a; p) = (a; q) = 1 \Rightarrow$
\begin{align}
\begin{cases}
a^{3p - 1} \equiv 1 \pmod{q} \\
a^{3q - 1} \equiv 1 \pmod{p}
\end{cases}
\end{align}
Chọn $a: ord_q(a) = q - 1 \Rightarrow (3p - 1) \vdots (q - 1)$.
Chọn $a: ord_p(a) = p - 1 \Rightarrow (3q - 1) \vdots (p - 1)$.
Ta có: $1 \le \dfrac{3p - 1}{q - 1} \le \dfrac{3(q - 2) - 1}{q - 1} < 3$ và $\dfrac{3p - 1}{q - 1} \ne 1$
$\Rightarrow \dfrac{3p - 1}{q - 1} = 2 \Leftrightarrow p = \dfrac{2q - 1}{3}$. Thay vào, tính được $(p; q) = (11; 17)$ (nhận).
Vậy $(p; q) = (11; 17); (17; 11) \blacksquare$
[RIGHT][I][B]Nguồn: MathScope.ORG[/B][/I][/RIGHT]
 

thay đổi nội dung bởi: sternritterp28, 25-02-2018 lúc 12:48 PM
sternritterp28 is offline   Trả Lời Với Trích Dẫn
Old 25-02-2018, 12:42 PM   #25
Thụy An
+Thành Viên+

 
Tham gia ngày: Oct 2017
Bài gởi: 93
Thanks: 1
Thanked 68 Times in 45 Posts
Trích:
Nguyên văn bởi sternritterp28 View Post
Chọn $a: ord_q(a) = 1 \Rightarrow (3p - 1) \vdots (q - 1)$.
Chọn $a: ord_p(a) = 1 \Rightarrow (3q - 1) \vdots (p - 1)$.
Cái này sai!
Trích:
Nguyên văn bởi namdung View Post
5. Tìm tất cả các cặp số nguyên tố $p;\, q$ sao cho $a^{3pq} \equiv a \pmod{3pq} $ với mọi số nguyên dương $a$.
Đây là bài Romania TST 1996, ý tưởng chủ đạo là sử dụng tiêu chuẩn Korselt cho số giả nguyên tố có bàn đến ở [Only registered and activated users can see links. ] . Để chứng minh tiêu chuẩn đó, ta cần sử dụng tính chất của cấp theo modulo và căn nguyên thủy.
[RIGHT][I][B]Nguồn: MathScope.ORG[/B][/I][/RIGHT]
 
Thụy An is offline   Trả Lời Với Trích Dẫn
The Following User Says Thank You to Thụy An For This Useful Post:
sternritterp28 (25-02-2018)
Old 25-02-2018, 12:48 PM   #26
sternritterp28
+Thành Viên+
 
Tham gia ngày: Aug 2015
Bài gởi: 4
Thanks: 1
Thanked 0 Times in 0 Posts
Trích:
Nguyên văn bởi Thụy An View Post
Cái này sai!

Đây là bài Romania TST 1996, ý tưởng chủ đạo là sử dụng tiêu chuẩn Korselt cho số giả nguyên tố có bàn đến ở [Only registered and activated users can see links. ] . Để chứng minh tiêu chuẩn đó, ta cần sử dụng tính chất của cấp theo modulo và căn nguyên thủy.
Mình bị nhầm chỗ này, cảm ơn bạn! Mình đã sửa trong bài bên trên rồi. (y)
[RIGHT][I][B]Nguồn: MathScope.ORG[/B][/I][/RIGHT]
 
sternritterp28 is offline   Trả Lời Với Trích Dẫn
Old 27-02-2018, 03:22 PM   #27
hoangleo963
+Thành Viên+
 
Tham gia ngày: Feb 2016
Đến từ: TP. HCM
Bài gởi: 5
Thanks: 6
Thanked 0 Times in 0 Posts
Trích:
Nguyên văn bởi namdung View Post
Đề 3 - Ngày 1, 22/2/2018

2. Cho tam giác $ABC$ nhọn, không cân nội tiếp đường tròn $(O)$. Gọi $BE, CF$ là hai đường cao và $H$ là trực tâm. Gọi $d$ là đường thẳng đi qua $H$ và song song với $BC$. Các đường thẳng đối xứng với $BE, CF$ qua $d$ cắt $AB, AC$ lần lượt tại $N, M$. $FM$ cắt $BE$ tại $X$, $EN$ cắt $CF$ tại $Y$. Chứng minh rằng đường tròn ngoại tiếp tam giác $HXY$ tiếp xúc với $(O)$ và đường tròn ngoại tiếp tam giác $HBC$.
Em là tác giả của bài toán này, sau đó do nhận thấy rằng đây là một bài thú vị trên mô hình tiếp xúc nhưng ẩn đi tiếp điểm nên em đã gửi cho thầy Nam Dũng. Khi khai thác sẽ còn nhiều tính chất khác để viết thành một bài toán nhiều ý như sau, dựa theo bạn theunknown:

Bài toán: Cho tam giác $ABC$ nhọn, không cân nội tiếp đường tròn $(O)$. Gọi $BE, CF$ là hai đường cao và $H$ là trực tâm. Gọi $d$ là đường thẳng đi qua $H$ và song song với $BC$. Các đường thẳng đối xứng với $BE, CF$ qua $d$ cắt $AB, AC$ lần lượt tại $N, M$. $FM$ cắt $BE$ tại $X$, $EN$ cắt $CF$ tại $Y$.

a) Chứng minh rằng đường tròn ngoại tiếp tam giác $HXY$ tiếp xúc với $(O)$ và đường tròn ngoại tiếp tam giác $HBC$.

b) Gọi $(HXY)$ tiếp xúc $(O)$ tại $Z$, $T$ là tâm của $(HXY)$, $EF$ cắt $BC$ tại $D$. Đường tròn $(DTZ)$ cắt $BC$ và $(O)$ tại $K,L$. $KL$ cắt $(O)$ tại $V$. Chứng minh rằng $AV\parallel BC$.

c) Gọi $AO$ cắt $BC$ tại $W$. Chứng minh rằng $TEWF$ nội tiếp.


Quanh bài toán này vẫn còn nhiều phát triển và mở rộng thú vị.
[RIGHT][I][B]Nguồn: MathScope.ORG[/B][/I][/RIGHT]
 
Hình Kèm Theo
Kiểu File : png Screenshot 2018-02-27 16.59.07.png (136.0 KB, 27 lần tải)

thay đổi nội dung bởi: hoangleo963, 27-02-2018 lúc 05:03 PM Lý do: Bổ sung hình vẽ
hoangleo963 is offline   Trả Lời Với Trích Dẫn
Old 01-03-2018, 07:21 AM   #28
buratinogigle
Administrator

 
buratinogigle's Avatar
 
Tham gia ngày: Jan 2016
Bài gởi: 50
Thanks: 57
Thanked 58 Times in 33 Posts
Trích:
Nguyên văn bởi namdung View Post
Đề 2 - Ngày 1.

1. Cho tứ giác $ABCD$ nội tiếp. Các điểm $M, N$ lần lượt di chuyển trên các cạnh $AD$ và $BC$ sao cho $ \frac{MA}{MD} = \frac{NB}{NC}$. Chứng minh rằng đường tròn đi qua giao điểm của các đường thẳng $AB, CD$ và $MN$ luôn đi qua hai điểm cố định khi $M, N$ thay đổi.
Em xin gửi đáp án cho bài toán này


Gọi giao điểm của $AB$ và $CD$ là $X$ thì $X$ cố định. Gọi giao điểm của $MN$ với $AB,$ $CD$ là $Y,$ $Z.$ Ta cần chứng minh đường tròn $(XYZ)$ đi qua một điểm cố định khác $X.$ Gọi giao điểm của $AD$ và $BC$ là $F.$ Đường tròn $(FAB)$ và $(FCD)$ cắt nhau tại $G$ khác $F.$ Ta dễ suy ra $\triangle GAD\sim\triangle GBC\ (g.g)$ mà $M,$ $N$ lần lượt chia $AD$ và $BC$ cùng tỷ số do đó $\triangle GAM\sim\triangle GBN\ (c.g.c).$ Từ đây suy ra tứ giác $GMNF$ nội tiếp. Từ đó $\angle GBA=\angle GFA=\angle GNM$ ta thu được tứ giác $GBNY$ nội tiếp. Vậy $\angle GYM=\angle GBF=\angle GXC$ (do tứ giác $GBCX$ nội tiếp theo tính chất điểm Miquel). Vậy $\angle GYZ=180^\circ-\angle GYM=180^\circ-\angle GXD=\angle GXZ$ từ đây ta có tứ giác $XYGZ$ nội tiếp, nói cách khác đường tròn $(XYZ)$ đi qua $G$ cố định khác $X.$
[RIGHT][I][B]Nguồn: MathScope.ORG[/B][/I][/RIGHT]
 
Hình Kèm Theo
Kiểu File : png Figure6750.png (43.8 KB, 125 lần tải)
__________________
Blog hình học sơ cấp [Only registered and activated users can see links. ]
buratinogigle is offline   Trả Lời Với Trích Dẫn
The Following User Says Thank You to buratinogigle For This Useful Post:
taikhoan2002 (01-03-2018)
Old 01-03-2018, 08:15 AM   #29
vuminhhieu
+Thành Viên+
 
Tham gia ngày: Feb 2018
Bài gởi: 1
Thanks: 0
Thanked 0 Times in 0 Posts
Có ngày thi TST chưa mọi người.
[RIGHT][I][B]Nguồn: MathScope.ORG[/B][/I][/RIGHT]
 
vuminhhieu is offline   Trả Lời Với Trích Dẫn
Old 01-03-2018, 08:35 AM   #30
supercht_no1
+Thành Viên+
 
Tham gia ngày: Feb 2016
Bài gởi: 2
Thanks: 2
Thanked 0 Times in 0 Posts

[RIGHT][I][B]Nguồn: MathScope.ORG[/B][/I][/RIGHT]
 
supercht_no1 is offline   Trả Lời Với Trích Dẫn
Trả lời Gởi Ðề Tài Mới

Bookmarks

Ðiều Chỉnh
Xếp Bài

Quuyền Hạn Của Bạn
You may not post new threads
You may not post replies
You may not post attachments
You may not edit your posts

BB code is Mở
Smilies đang Mở
[IMG] đang Mở
HTML đang Tắt

Chuyển đến


Múi giờ GMT. Hiện tại là 03:58 PM.


Powered by: vBulletin Copyright ©2000-2024, Jelsoft Enterprises Ltd.
Inactive Reminders By mathscope.org
[page compression: 113.94 k/130.29 k (12.54%)]